Download as pdf or txt
Download as pdf or txt
You are on page 1of 69

DEDAN KIMATHI UNIVERSITY OF TECHNOLOGY

SCHOOL OF SCIENCE

DEPARTMENT OF MATHEMATICS AND PHYSICAL SCIENCES

COURSE UNIT: SPH 1120 PHYSICS I

LECTURE I: LANGUAGE OF PHYSICS

I.1 DEFINITIONS

Physics is science dealing with the most general and simplest proprieties and forms of motion of matter. It deals with quantitative
relationships between physical quantities of different objects and their interactions. Matter: all what exists objectively, independently of
anyone consciousness, sensations or will.

There are two forms of matter: substance and field. Substance includes atoms, molecules, elementary particles and all bodies built of them.
Electromagnetic, gravitational and other fields form the second form of matter.

Matter is in continuous motion which means change in general.

Motion is an unalienable propriety of matter. Matter exists and moves in space and time. There is nothing out of space or out of time

Physics is an experimental science, meaning laws of physics are established by generalizing experimental facts. They express the objective
regularities existing in nature. The goal of physics is to find the most basic laws that govern the universe and to formulate those laws in the
most precise way possible. For this, these laws are expressed in the form of quantitative relationships between various physical quantities.

Therefore, physics is a mathematical science and the general language of physics is mathematics: laws of physics are generally expressed
in the form of mathematical expressions, using symbols that represent different physical quantities

A physical quantity is any measurable propriety of an object. To measure a physical quantity is to compare that quantity with another one of
the same kind taken as unit.

I.2. INTERNATIONAL SYSTEM OF UNITS

The requirements of industry, commerce and science dictate that there shall be standard units internationally agreed upon.

The International System of Units (abbreviated SI from French: Le Système International d'Unités) is the modern form of the metric
system which is a decimal system (based on power of ten). It is a coherent system of units of measurement built around seven base units, 22
named and an indeterminate number of unnamed coherent derived units, and a set of prefixes that act as decimal-based multipliers. The
standards, published in 1960, are based on the metre-kilogram-second, MKS, system, rather than the centimetre-gram-second, CGS,
system, which, in turn, had several variants.

The SI has been declared to be an evolving system; thus prefixes and units are created and unit definitions are modified through international
agreements as the technology of measurement progresses, and the precision of measurements improves. SI is the world's most widely used
system of measurement, used in everyday commerce , science and technology.

The international organ responsible for the SI system is the General Conference on Weights and Measures and its subsidiaries

In this system, the three fundamental physical quantities are: Length (L), Mass (M) and Time (T)

1
I.2.1. STANDARD SI UNITS

The standard unit for Length (L) is the metre (m), the standard unit for Mass (M) is the kilogram (kg) and the standard unit for Time (T) is
the second (s). Table I.1 gives the different base units of SI system

Table I.1 Table of base units of SI

Physical quantity Name of SI unit Symbol

Length metre m

Mass kilogram kg

Time second s

Electric current Ampere* A

Thermodynamic or absolute temperature Kelvin* K

Luminous Intensity candela cd

Amount of substance mole mol

*André-Marie Ampère (20 January 1775 – 10 June 1836) was a French physicist and mathematician who is generally regarded as one of
the main founders of the science of classical electromagnetism, which he referred to as "electrodynamics". The SI unit of measurement of
electric current, the Ampere, is named after him.

**William Thomson, 1st Baron Kelvin (26 June 1824 – 17 December 1907) was an Irish-born British mathematical physicist and
engineer. At the University of Glasgow he did important work in the mathematical analysis of electricity and formulation of the first and
second laws of thermodynamics, and did much to unify the emerging discipline of physics in its modern form. For his work on the
transatlantic telegraph project he was knighted by Queen Victoria, becoming Sir William Thomson Kelvin

The mole

All molecules or atoms of a pure substance are supposed to be absolutely identical and have the same size. Their mass is very small. The
mass of 1 atom of the isotope of carbon-12 is 19.92 X 10 −27 kg . The number of particles N in a sample of a pure substance of total
mass M is:

M
N= (I.1),
m
where m is the mass of a single particle
The “mole” is the amount of a substance or of a system which contains 6.02 X1023 elementary entities. This number is called Avogadro
number, one of the universal physical constants

*Lorenzo Romano Amedeo Carlo Avogadro di Quaregna e di Cerreto, Count of Quaregna and Cerreto (9 August 1776 – 9 July 1856) was
an Italian savant. He is most noted for his contributions to molecular theory, including what is known as Avogadro's law. In tribute to him,
the number of elementary entities (atoms, molecules, ions or other particles) in 1 mole of a substance, 6.02214179 X 10 23 , is known as
the Avogadro constant.

I.2.2. DERIVED SI UNITS

The units of physical quantities other than mass, length, time, current… are combinations of the base units and are called derived units

2
For example, the unit of speed is
m
, we write: v = m = m.s −1 , for acceleration, m
or a  = m
2
kg
= m.s − 2 and for density 3 .
s s s2 s m
or D  = kg
= kg.m −3
m3
Sometimes, a derived unit is given a special name to honour someone. These are called “named units”

For example, the SI unit of force is found according to the equation F = ma → F  = ma , →

F  = kg m2  = kg m2 .
s  s

The SI unit for force is called “Newton, N ” in the honour of Sir Isaac Newton* the founder of classical mechanics.

We write: F  = kg.m.s −2  N
*Sir Isaac Newton PRS MP (25 December 1642 – 20 March 1727 [NS: 4 January 1643 – 31 March 1727]) was an English physicist,
mathematician, astronomer, natural philosopher, alchemist and theologian, who has been "considered by many to be the greatest and
most influential scientist who ever lived." His monograph Philosophiæ Naturalis Principia Mathematica, published in 1687, lays the
foundations for most of classical mechanics. In this work, Newton described universal gravitation and the three laws of motion, which
dominated the scientific view of the physical universe for the next three centuries. Newton showed that the motion of objects on Earth and
that of celestial bodies are governed by the same set of natural laws, by demonstrating the consistency between Kepler's laws of planetary
motion and his theory of gravitation, thus removing the last doubts about heliocentrism and advancing the Scientific Revolution.

I.2.3. IMPERIAL UNITS

Imperial units were used in Britain and the Commonwealth countries. In the United States metric units are not commonly used outside of
science, medicine and the government; however, United States Customary Units are officially defined in terms of SI units. The following
example shows how conversion of Imperial Units to SI units can be done.

Practical Activity I.1

Example I.1

Conversion of Imperial units to SI units

Vesna Vulovic survived the longest fall on record without a parachute when her plane exploded and she fell 6 miles or 6mi , and
551 yards or 551yd . What is this distance in metres?

Solution

We use the facts that 1mi = 5280 ft , 1m = 3.281 ft , and 1yd = 3 ft , where mi represents mile, yd represents yard and ft is
(5280 ft ) = 1 , (1m ) = 1 and (3 ft ) = 1→ By multiplying by the
(1mi) (3.281 f )t
foot. With these facts we construct three conversion factors:
1 yd
given distance d of the fall by the appropriate conversion factors we find that
 5280 ft  1m   3 ft  1m 
d = (6mi)   + (551yd )   = 10159m
 1mi  3.281 ft   1yd  3.281 ft 

Example I.2

3
 mi 
Bicyclists in the Tour de France reach speeds of 34.0 miles per hour   on flat sections of the road. What is this speed in (a)
 h 
 km  m
kilometers per hour   and (b) metres per second   ?
 h  s

Solution

 mi   km 
(a) To convert the speed from miles per hour   to kilometres per hour   , we need to convert miles to kilometres. This
 h   h 
conversion is achieved by using the relation 1.609km = 1mi
 mi  m
(b) To convert the speed from miles per hour   to metres per second   , we must convert miles to metres and hours to
 h  s
seconds. This is accomplished by using the conversions 1mi = 1609m and 1h = 3600s →

mi  1.609km 
(c) Multiplying the speed of 34.0   = 1 , we find the speed of the bicyclists is:
by a factor of unity,
h  1mi 
 mi   mi  1.609km 
v =  34.0 (1) =  34.0 
km
 = 54.7
 h   h  1mi  h

mi  1609m  (1h) = 1 , the speed of the bicyclists is:


34.0   = 1 and
(3600s )
(d) Multiplying the speed of by two factors of unity,
h  1mi 
 mi   mi  1609m  1h 
v =  34.0 (1)(1) =  34.0 
m
  = 15.2
 h   h  1mi  3600 s  s

In the course we will use SI units only, unless specifically stated otherwise!!!!!

I.3. DIMENSIONS

The technique of dimensions is commonly used in mechanics and electromagnetism to check the “dimension” of a physical quantity or
establish the relationship between SI derived units. This is important because ”an equation which is “dimensionally wrong” or “not
dimensionally consistent” is “absolutely wrong”!!!!

Mechanical measurements were at first based on three base measures: length, mass and time. As science developed, other fundamental
measures were added.

It is common in physics literature to find such statement as “velocity has dimension LT


−1
”. This is written: v = LT −1
This statement means:

“Independently of the system of units used, the measurement of velocity requires the numerical value of a distance (length) be divided by
the numerical value of a time”

Examples: Velocity: velocity =


length
or v = L = LT −1
time T

Volume: Volume = length 3 or V  = L3

4
Density: Density =
mass
or D = ML−3
volume

where M = mass , L  length and T  time

We know that F = ma is the equation which gives the numerical value of a force. Dimensionally, we may write:

d
F  = ma , acceleration, v d
a= = t = 2 → a = LT −2 → F  = MLT −2
t t t
Incidentally, this equation provides us with the unit of force:

F  = MLT −2  kg.m.s −2  Newton  N


We can employ the “method of dimensions” or “dimensional analysis” to check the validity of expressions and equations and even to
predict the form of equations

The ability to do this depends on the necessity that all dimensions on each side of an equation must be the same.

Example: the period of oscillations of a simple pendulum is given by:


T = 2 (I.3)
g

The left side has the dimension of time, T :

1
  L2
1

1

  = = L2 L 2 T = T
( )
1
 g  LT − 2 2

We’ll say that equation (1.3) is “dimensionally consistent”

The dimensional analysis can even help us in finding the correct expression of a physical quantity.

I.4 STANDARD FORM OF NUMERICAL VALUES

In physics, we deal with some numbers that are very small, e.g. the radius of an atom and others that are very large, e.g. the mass of the
Earth. Therefore we prefer to adopt a standard form when writing numerical values which is based on “power of ten “ notation. For example
a book of data will quote the coefficient of linear expansion of aluminium as:  Al = 2.3 X 10 −6 K −1 rather
than  Al = 0.0000023K −1 .

There is a preference for the use of indices:  3,  6 ,  9 …

The method uses indices to represent powers of 10 when the zeros are already counted:

→ 0.000,000,001 = 1X 10 −9 and 1,000,000,000 = 1X 10 9

In practice any number is represented in such a way that we have only one digit in front of the decimal point, the rest of the digits also called
the mantissa, is placed to the right side of the point. The order of magnitude is expressed by the index of power of 10.

Examples: 0.000098  9.8 X 10 −5 and 1189  1.189 X 10 3


5
−5
In these examples the quantities 10 and 10 3 is called “order of magnitude” of the respective quantities

I.5. PREFIXES OR MULTIPLES AND SUBMULTIPLES OF SI UNITS

To express values which are very small or very big parts of a unit, prefixes or multiples and submultiples of the SI units are used according
Table 0.3.

Table I.3 Table of prefixes for SI units

Submultiple Prefix Symbol Multiple Prefix Symbol

10 −3 milli m 10 3 kilo k

10 −6 micro  10 6 Mega M

10 −9 nano n 10 9 Giga G

10 −12 pico p 1012 Tera T

10 −15 femto f 1015 Peta P

10 −18 atto a 1018 Exa E

For 1.05nF = 1.05 X 10 −9 F and 300MHz = 3 X 10 2 X 10 6 Hz = 3 X 10 8 Hz ,


example: where 1F and 1Hz mean
1Farad and 1Hertz and are SI units for capacitance and frequency, respectively

Practical Activity I.2

Example I.3
−6
The mass of the parasitic wasp Caraphractus cintus can be as small as 5 X 10 kg . What is this mass in (a) grams, g , (b)
milligrams, mg , and (c) micrograms g ?

Solution

When converting between units, we write down the units explicitly in the calculations and treat them like any algebraic quantity. We
construct the appropriate conversion factor (equal to unity) so that the final result has the desired units.
(a) Since one thousand grams = 1.0 kilogram: 1X 10 3 g = 1.0kg , it follows that the appropriate conversion factor is

 1.0 X 10 3 g 
  = 1
 1.0kg 
 1.0 X 10 3 g 
Therefore (5 X 10 −6
)
kg   = 5 X 10 −3 g
 1 .0 kg 

(5 X 10 g ) 1.0 X1.10 mg  3


(b) As one thousand milligrams = 1 gram: 1X 10mg = 1.0 g →
−3
 = 5.0mg
 0g 
g 
(5 X 10 g ) 1.0 X1.10
6
(c) Similarly: 1.0 X 10 6 g = 1.0 g →
−3
 = 5 X 10 3
g
 0g 
6
I.6 REFERENCE FRAMES AND COORDINATE SYSTEMS

The position of a body in space has meaning only in relation to a frame of reference or reference frame which is sometimes physical, such
as a tabletop, a room, a ship or the Earth itself or just imaginary. In a reference frame, the position of an object is specified with respect to a
coordinate system that consists of a point taken as origin, O, and a set of three non coplanar axes, each of which specifies a direction in
space. The time is indicated using a time piece

In the Cartesian* coordinate system, the axes are labeled X,Y and Z . These straight lines are mutually perpendicular and intersect at the
origin O . In two dimensions, a point P may be located by its Cartesian coordinates (x, y ) as shown in Fig. I.2. A scale is marked on each
axis. Starting at O , x and y are the number of units ( + or − ) one must move in the direction of each axis to reach P .

In plane polar coordinates, the length of the line OP and the angle  with respect to the reference direction ( + X axis) are given. These
two types of coordinates are related as follows:

x = r cos (I.4a)

y = r sin  (I.4b)

where

r= x2 + y2 (I.5a),

y
tan  = (I.5b)
x

Note that  is measured counterclockwise from the + X axis

Fig.I.2. Cartesian coordinates of point P are (x, y ) . Its polar coordinates are (r , )

*The adjective “Cartesian” comes from the name Descartes**.

**Rene Descartes (31 March 1596 – 11 February 1650). He is credited as the father of analytical geometry, the bridge between algebra
and geometry, and he was crucial to the discovery of infinitesimal calculus and analysis founded by I. Newton

1.7. VECTORS AND SCALAR QUANTITIES

In physics one can distinguish quantities which are determined by a direction beside a magnitude, these quantities are called ”vector
quantities” and others which are determined by a magnitude only. These are called scalars. Mass, work, energy, current intensity… are
scalars.

7
Force, velocity, acceleration, electric field intensity… are vectors. Vectors obey laws of vector algebra. Particularly, addition of vectors
applies the parallelogram rule of addition as shown in Fig.I.3

Fig.I.3. Parallelogram rule of addition and subtraction of vectors

We will see more on vectors in Lecture II

I.8. SUBDIVISION OF PHYSICS

Physics is a very wide subject. Between 1600 and 1900, three broad areas were developed in what is now called classical physics:

1. Classical mechanics: The study of motion of particles and fluids

2. Thermodynamics: The study of temperature, heat transfer and the proprieties of aggregations of many particles. It is the study of heat as
one of forms of energy

3. Electromagnetism: Electricity, magnetism, electromagnetic waves and optics

These three areas encompass virtually all the physical phenomena which we are familiar with.

However, by 1905 it became apparent that classical ideas failed to explain several phenomena, especially when it comes to atoms, their
structures and their interaction with radiation. Three important theories in modern physics are:

4. Special relativity: A theory of the behaviour of particles moving at high speeds. It led to a radical revision of our ideas of space, time
and energy

5. Quantum mechanics: A theory of the submicroscopic world of the atom. It also required a profound upheaval in our vision of how
nature operates

6. General relativity: a theory that relates the force of gravity to the geometrical properties of space

The applications of these theories are:

7. Atomic physics: study of the structure of atoms and their interaction with radiation

8. Nuclear physics: study of structure and transformations of atomic nuclei

I.9. PHYSICS AND ENGINEERING

Many people ask such a question: “I’ve have applied for a BSc in engineering, be mechanical, telecommunications, civil, or electrical, why
hell do I need physics”? The formal answer is simple: the role of an engineer is to transform nature (matter) on the benefit of mankind;
physics is about general motion of matter and its interactions. Therefore training of an engineer must start with physics. One could also say
that engineering is simply application of physics because all engineering techniques and technologies are based on laws of physics

8
To illustrate this here is given a table of several topics in engineering with the reference to the course of physics to give an idea of how
crucial physics is for an engineer

Table I.4. Physics and Engineering

Topic Area of engineering Topic in physics Law of physics

Strength of materials Mechanical, structural, Elasticity, statics Hooke’s law


electrical

Theory of mechanisms and Mechanical Rotational dynamics, statics, Newton’s law of rotational motion
machines simple machines

Electric power generation Electrical Electricity and magnetism Faraday’s law of electromagnetic
induction

Electric machines Electrical Electricity and magnetism Electrical motor principle

Thermal machines Mechanical Thermodynamics First and second laws of


thermodynamics

Power transmission Electrical Electricity and magnetism Ohm’s law, Joule-Lenz law

Wireless communications Telecommunications Electricity and magnetism Maxwell’s equations

TV and radiobroadcasting Telecommunications Electricity and magnetism Maxwell’s equations

Optical fibre communications Telecommunications Optics Maxwell’s equations, total internal


reflection

Water and sanitation Civil Fluid mechanics Bernouilli equation and related
laws

Roads and bridges Civil Mechanics, statics Elasticity, conditions of dynamic


equilibrium

The list is endless

I.10. SOME SIMPLE GUIDELINES ON HOW TO SOLVE PROBLEMS IN PHYSICS

1. Understand the problem: Read the problem again and again until you are sure you fully understand the problem. For this, try to
reformulate the problem using your own words. A small sketch can be quite handy!!! If you are unable to understand the problem
don’t waste time! Go to the next question!!!

2. Numerical data: If the problem is well understood, write down the numerical data in SI units using appropriate symbols for
physical quantities in standard form and in order of appearance of different data in the problem

3. Solution: By reasoning, try to find (in your mind) a general expression which related the quantity to be found, the unknown, with
the quantities given in numerical data. This the heart of process

AB
4. Working equation: Find the “working equation” of the form: x = , where x is the unknown or the quantity to be found.
D
The right-hand side of the equation should contain only constants and symbols of quantities given in “numerical data”. As much as
possible, avoid performing intermediate calculations, especially for quantities you are not asked to find!!!!!

5. Dimensions: Check the dimensions of the working equation, the dimension of the left-hand side and the right-hand side must
coincide and coincide with the dimension of the unknown. Remember: an expression that is “dimensionally wrong is absolutely
wrong”!!!!!
9
6. Numerical application: Replace the numerical values of different quantities appearing in the working equation to calculate the
numerical value of x i.e. the answer. When the dimension have been checked, it is not necessary to include the units in the
numerical expression. When calculating, calculate the numerical coefficients together and the order of magnitude (powers of ten)
together. Use always the standard notation!!!

7. The last, but not the least!!! Ask yourself this simple question “Is my answer reasonable??” !! It can be a matter of the order of
magnitude or other logical reasons that make your answer unreasonable. If you get an unreasonable answer please, repeat the
solution of the problem! Never present an unreasonable answer!!!!

Review and summary


Physics is the study of general forms of motion of matter. Matter is in continuous motion in time and space

The methodology of physics is generalisation of experimental results on relationships between physical quantities of different objects in
interaction. These relatioships are generally expressed in a mathematical form. Thus mathematics is the language of physics.

A value of a physical quantity is expressed in terms of a numerical value and a unit of measurement. The base mechanical physical
quantities are mass M , length L and time T . The SI unit of mass is the kilogram, kg : M  = kg , the unit of length is metre,

m : L = m and the SI unit for time is second, s : T  = s

Other physical quantities are expressed using derived units that are combinations of base units.

For clarity and simplification of calculations, values of physical quantities are represented using scientific or standard notation based on
power of ten and prefixes of SI units.

To monitor motion of objects in space and time, we use a reference frame which consits of an origin point, a system of coordinates and a
timepiece.

Physical quantities can be classified into two categories: scalars are quantities that are fully defined by a magnitude and vectors are
qunantities that are defined by a magnitude and an orientation in space. Vectors obey law of vector algebra

Physics is extremely important for engineers because all engineering techniques are based on laws of physics

Practical Activity I.4

Questions and problems

g
(1) The density of water is about 1 . What is this in SI units?
cm 3
m
(2) (i) The distance traveled by light in a year is called a light-year. Given that the speed of light is c = 3.00 X 10 8 ,
s
express the light-year in km
(ii) The distance between the Earth and the Sun is called an astronomical unit, AU, and its value is about
AU
1AU = 1.5 X 1011 m . What is the speed of light in ?
h
(3) Specify the number of significant figures in each of the following values: (i) 23.001 (ii) 0.500 X 10 2 m (iii)
kg
0.002030kg (iv) 2700
m3
6.5ns (b) 12.8m (c) 20,000 MW (d) 0.003mA (e) 1.5 pA
(4) Express the following values in the SI units: (a)
(5) Given that  = 3.14159 , find: (a) the area of a circle of radius R = 4.2m , (b) The surface area of a sphere of
radius R = 0.46m , (c) the volume of a sphere of radius R = 2.318m

10
(8.00 X 10 )
1
1.002 6 −3
(6) Express the following numbers in standard notation: (i) (ii) (iii) 0.0000076300
4.0

(7) Evaluate: (i)


(3.00 X 10 )(1.2 X 10 )
12 −12
(ii) 1.075 X 10
2
− 6.37 X 10 + 4.18 (iii) 402.1+1.073
(4.18 X 10 )
1
−1 − 2

(8) Convert the following to scientific notation:

(i) the distance to the Sun d = 149,500,000,000m


(ii) the wavelength of yellow sodium light:  = 0.000,000,5893m
(iii) the radius of an atom: r = 0.000,000,000.2m
(iv) the radius of a nucleus: r = 0.000,000,000.000,004m

(9) (i) What is the surface area of the Earth (ii) What is the volume of the Earth (iii) the mass of the Earth

(10) A watch is advertised as being 99% accurate. Would you buy it?

(11) How fast is a person at the Equator moving relative to the a person at the North Pole

(12) How many frames are there in a 2h - film: (what information do you need?

(13) Use a ruler to estimate the thickness of a sheet of paper in a book

(14) How many litres of water would be need to raise the level of Lake Victoria by 25cm . What information do you need?

(15) Supposing a body of mass m on the Earth’s surface, according the Newton’s second law of dynamics: F = ma the
mM
force of gravity Fg = mg and according the Newton’s law of gravitation : Fg = G 2 , where M , the mass of
R
the Earth

(i) Find the dimension of quantity G called ” gravitational constant”


(ii) Express the acceleration due to gravity g through Earth’s parameters

(iii) Since numerical value in SI for G = 6.67 X 10 −11 and the radius of the Earth R = 6,400km , calculate
the mass of the Earth

11
LECTURE II. VECTORS
LECTURE OUTLINE

In this lecture, we will study simple mathematical operations used over vectors because they do not obey laws of ordinary algebra and again
they are so important in physics. We will define the sum, difference of two vectors, their proprieties and representation. We will see that there
are two types of products of vectors: scalar and vector product. These mathematical operations reflect real physical quantities, such work,
flux, and torque, force in a magnetic field for the scalar and vector product, respectively

II.1 VECTORS AND SCALARS

Definition: A vector quantity is physical quantity that has both a magnitude and a direction in space and thus can be represented by a vector.

The simplest vector quantity is displacement or change in position. A vector that represents a displacement is called a displacement vector.
If a particle changes its position by moving from A to B like in Fig. II.1, we say it undergoes a displacement from A to B, which we represent
with an arrow that points from A towards B. The arrow specifies the vector graphically:

- the length of the arrow represents the magnitude on a certain scale


- the orientation in space of the arrow represents the direction

In Fig.II.1 (a), the arrows from A to B, from A/ to B/, and from A// to B// have the same magnitude and direction: they specify identical
displacement vectors and represent the same change in position for the particle. A vector can be shifted parallel to itself without changing its
value, if its magnitude (graphically its length) and its direction are not changed

Notice that vector tells nothing about the actual path that the particle follows to get to B from A. In Fig.II.1 (b) all the three paths connecting
A and B correspond to the same displacement vector.

Not all physical quantities involve a direction in space. Temperature, pressure, mass, energy and time for example do not “point” in any
direction of space. Such quantities are called scalars and are represented by a magnitude: a numerical value and a unit. We deal with them
using ordinary algebra

II.2. ADDING VECTORS GEOMETRICALLY

Suppose that as in vector diagram of Fig.II.2 (a), a particle moves from A to B and then to C. We can represent its overall displacement, no
matter what its actual path, with two successive displacements AB and BC. The net displacement is a single displacement from A to C. We
call AC the vector sum or the resultant of vectors AB and BC. The vectors are redrawn in Fig.II.2 (b) and label them in the way that we shall
use from now on: with an arrow or a bar over an italic letter or letters: a or AB . To indicate only the magnitude we can use the
symbols, a or AB and a or AB will be represented. In textbooks, where the bold facility is available, vectors are usually represented by
bold letters

The relation among the three vectors in Fig.II.2 (b) can be written in the form of a vector equation:

s = a + b or AC = AB + BC (II.1),

which says that vector s is the vector sum or the resultant of vectors a and b .
In the parallelogram rule, the vector sum or resultant of the two vectors is one of the diagonals of the parallelogram constructed with two
vectors as sides and the other diagonal is the difference between them as shown in Fig.II.2 (a). This is called the parallelogram rule.

The same result is obtained by using triangle rule or the tail-to-tip method. In this method, the tail of each vector is placed at the tip of the
preceding one. The resultant is drawn from the tail of the first vector to the tip of the last vector regardless of the number of vectors to be
added.

12
Fig.II.1 (a) All three arrows represent the same displecement (b) Displacement does not consider the real path of the particle

Vector addition has two important proprieties:

(i) it is commutative:

a+b = b+a (II.2)

(ii) It is associative:

(a + b) + c = a + (b + c ) (II.3)

The two proprieties are represented in Fig.II.3 and Fig.II.4

(a)

(b)

Fig.II.2 (a) Vector addition is commutative (b) Vector addition is associative

13
In Fig.II.2 (a) both the parallelogram and triangle rules are represented.

The vector ( ) is a vector with the same magnitude as b but the opposite direction (see Fig.II.3)
− b

Fig.II.3. Vector b and vector − b()


Adding the two vectors yields:

( )
b+ −b = 0 (II.4)

Thus adding − (b ) has the effect of subtracting b . Then

d = a − b = a + (− b) (II.5)

This is graphically shown in Fig.II.4 (a) and (b)

Fig.II.4. Subtraction of vectors

II.3. COMPONENTS OF VECTORS AND UNIT VECTORS

II.3.1. COMPONENTS OF VECTORS

Adding vectors geometrically can be tedious. Sometimes it is easier to use components of vectors along the coordinate axes. A component of
a vector is the projection of the vectors on an axis. In Fig.II.5 (a) a x is the component of vector a along x -axis and a y is the component of
vector a along y -axis

We see that

Ax = A cos  , Ay = A sin  (II.6)

The magnitude and direction in terms of components are:

14
A= Ax + Ay
2 2
(II.7)

Ay
tan  = (II.8)
Ax

Fig.II.5 Cartesian or rectangular components of vector A are Ax and Ay

Practical Activity II.1

Example II.1

A man walks 5m at 37 0 North of east and then 10m at 60 0 West of North. What is the magnitude and direction of his net
displacement?

Solution

The x and y axes in Fig.II.6 point East and North respectively. We label the first displacement A and the second one B and the resultant

Fig.II.6. The resultant of vector A and B is vector R

The components of R are:


15
R x = Ax + B x = 5 cos 37 0 − 10 sin 60 0 = −4.66m

R y = Ay + B y = 5 sin 37 0 + 10 cos 60 0 = +8.00m

The magnitude is:

R = R x + R y = 9.26m
2 2

The direction is given by:

Ry + 8.00
tan  = = = −1.72
Rx − 4.66

The angle could be either 120 0 or − 60 0 but as we see in Fig.II.4  = 120 0 . The net displacement is 9.26m at 30 0 West of North

I.3.2. UNIT VECTORS

To simplify the manipulation of vectors, it is convenient to introduce the unit vectors i, j and k

that lie along the x, y and z axes, respectively.

A unit vector is a dimensionless quantity that serves only to specify a direction in space. It has unit length: i = y = k =1.

In Fig.II.7, the unit vectors are drawn from the origin for convenience

Fig.II.7. Unit vectors along corresponding axes

Any vector may be expressed as the sum of three vectors parallel to each of the coordinates axes as shown in Fig.II.8:

A = Ax i + Ay j + Az k (II.9)

We see that the magnitude of the vector is given by:

A= Ax + Ay + Az
2 2
(II.10)

In unit vector notation, the equality A = B is written as: Ax i + Ay j + Az k = Bx i + B y j + Bz k

16
Fig.II.8. Representation of a vector by its components

Since unit vectors i, j and k are mutually perpendicular, this means that

Ax = B x , Ay = B y , Az = Bz

“ If two vectors are equal, then each of their components must be equal and vice versa”

The vector equation R = A+ B  Rx = Ax + Bx , R y = Ay + B y and Rz = Az + Bz

The difference between the two vectors:

S = A − B = ( Ax − Bx )i + (Ay − B y ) j + ( Az − Bz )k

Practical Activity II.2

Example II.2

Given the vectors A = 2i − 3 j + 6k , m and B = i + 2 j − 3k , m , find (a) A + B (b) A+ B (c) 2 A − 3B

Solution

(a) The sum of the magnitudes of the two vectors:

A = 2 2 + 3 2 + 6 2 = 7.00m , B = 12 + 2 2 + 3 2 = 3.74m

→ A + B = 7.00m + 3.74m = 10.74m

(b) A + B = (2 + 1)i + (− 3 + 2) j + (6 − 3)k = 3i − j + 3k

→ A + B = 3 2 + 12 + 3 2 = 4.26m

Here we see clearly that A + B  A+ B

(b) ( ) ( )
2 A − 3B = 2 2i − 3 j + 6k − 3 i + 2 j − 3k = i − 12 j + 21k
17
II.4. SCALAR OR DOT PRODUCT OF VECTORS

In physics there are quantities that involve the product of two vectors. Such a quantity is work, W . This has led to the definition of two types
of products of vectors. The first one is the scalar product the other one is the vector product.

The scalar or dot product of two vectors A and B is defined as:

A.B = A . B cos  (II.11),

where A and B are the magnitudes of two vectors and  the angle between them

The scalar product is commutative: A.B = B. A


The scalar product of the unit vectors:

i.i = j. j = k .k = 1 (II.12)

i. j = j.k = i.k = 0 (II.13)

The scalar product is distributive:

(A + B ).C = A.C + B.C (II.14)

Now using (II.12) and (II.13), we get:

( )(
A..B = Ax i + Ay j + Az k. . Bx i + B y j + Bz k ),
that becomes

A.B = Ax B x + Ay B y + Az B z (II.15)

Practical Activity II.3

Example II.3

Find the angle between A = 2i + j + 2k and B = 4i − 3 j

Solution

From expression (II.12) and (II.15),

A.B. 2 X 4 − 1X 3 + 0 1
cos  = = = →  = 70.5 0
A. B 3X 5 3

II.5. VECTOR OR CROSS PRODUCT

The vector or cross product of two vectors A and B is defined as

AX B = A . B sin  .n (II.16)

18
The product is defined to be a vector of magnitude A . B sin  that points in the direction of the unit vector n normal to the plane of

A and B . The angle  is the smaller angle between the vectors.

The definition of the cross products reflects the behavior of many physical quantities such as torque, angular momentum, force in a magnetic
field…

To know the orientation of the vector product we use the right-hand rule as explained in Fig.II.9

Fig.II.9. The right-hand rule for the vector product

Curl the fingers of your right hand and stick out your thumb as if you were hitch-hiking. The sense of rotation of the fingers should be from
the first vector A to the second vector B -through the smaller angle between them. The thumb indicates the direction of n .
The vector product is anticommutative:

B X A = − AX B (II.17)

The vector product is distributive:

(A + B )X C = AX C + BX C (II.18)

In order to define the vector product, including the right-hand rule to be applied to unit vectors i, j and k , the coordinate system must be
right-handed.

Then

iXi = 0, jXj = 0, kXk = 0, (II.19a)

iXj = k , jXk = i, kXi = j , (II.19b)

The general expression for the vector product C = AXB :

19
ijk
( ) (
C = AX B = Ax i + Ay j + Az k. X Bx i + B y j + Bz k = Ax Ay Az ) (II.20)

Bx B y Bz

or

C = (Ay Bz − Az B y )i + ( Az Bx − Ax Bz ) j + (Ax B y − Ay Bz )k (II.21)

In expression (II.20), the operation between the long straight bar is called a determinant.

Practical Activity II.3

Example II.3

Find the vector product of A = 3i − 2 j + k and B = i + 4 j − 2k

Solution

AX B = (3i − 2 j + k )X (i + 4 j − 2k ) = (12k + 6 j ) + (2k + 4i ) + ( j − 4i ) = 7 j + 14k

AX B = 7 j + 14k

Review and summary


In physics, there are two categories of physical quantities: scalars and vectors

A scalar quantity is fully determined by a magnitude: numerical value and unit

A vector quantity is determined by a magnitude and an orientation in space

To represent a vector we use a line segment whose length represents the magnitude on a certain scale and the orientation corresponds to the
orientation of the quantity.

To add and subtract vectors, the parallelogram and triangle rules are used. When adding many vectors, the tail-to-tip method as shown in
Fig.II.10 is the fastest.

Fig.II.10. Tail-to-tip method of addition of vectors: R = A+ B +C

In two dimensions the rectangular or Cartesian components of the vector A of Fig. II.11

are: Ax = A cos  , Ay = A sin 

20
Fig.II.11. The x and y components of A are Ax and Ay

The magnitude and direction of the vector may be related to its components:

Ay
A= Ax + Ay tan  =
2 2
,
Ax

The angle  is measured from the + x axis moving anticlockwise.


In three dimensions a vector may be expressed in unit vector notation:

A = Ax i + Ay j + Az k

Its magnitude is:

A= Ax + Ay + Az
2 2 2

The vector equation R = A + B is equivalent to three equations:

Rx = Ax + Bx R y = Ay + B y Rz = Az + Bz

The scalar (dot) product of two vectors:

(A.B) = A B cos = A B x x + Ay B y + Az Bz

The vector (cross) product is

AX B = A . B sin  .n ,

Where direction of n is given by the right-hand rule.

Self-test II
II.1. True or false: (i) The components of a vector depend on the choice of coordinate system

(ii) The magnitude of a vector depends on the choice of coordinate system

(iii) The magnitude of a vector cannot be negative

21
(iv) If A− B = C − D  A = C and B = D

II.2. Which of the following are valid statements: (i) A = 5m

(ii) B = −10km

(iii) A+ B = C

(iv) (A.B ) = C ?
II.3. Draw a vector diagram to illustrate each of the following results for A+ B (i) 0 (ii) A + B

(A )
1
2 2 2
(iii) A−B (iv) +B

II.4 Can the magnitude of the difference of two vectors be greater than the magnitude of their sum? If so, illustrate with a vector
diagram.

II.5 Describe two vectors A and B such that:

(i) A + B = C and A + B = C

(ii) A+ B = A− B
2 2 2
(iii) A + B = C and A + B = C
II.6. Which of the arrangements of axes in Fig.II.12 can be labeled “right-handed coordinate system”? As usual each axis label indicates
the positive side of the axis

II.7. If A = 2i + 4 j , what is AX B when (i) B = 8i + 16 j , (ii) B = −8i − 16 j and (iii) B = 8i − 16 j

22
Practical Activity II.4

Exercises II

II.1. The magnitude of the vectors A and B shown in Fig.II.13 are A = 3m and B = 4m . Find the graphically: (i) A + B
(ii) A− B = C

II.2. The resultant of two vectors A and B is 40m due North. If A is 30m in the direction 30 0 S of W, find B graphically.

II.3. Three vectors are specified as follows: A is 5m at 45 0 N of E, B is 7m at 60 0 E of S and C is 4m at 30 0 W of S. Find the


magnitude and direction of their sum.

II.4. The resultant R of two displacements is 10m at 37 0 W of N. If the second displacement was 6m at 530 N of E, what was the
first?

C
II.5. Given two vectors A = 2i − 3 j + k and B = −4i + j − 5k , find a third vector C such that A − 2B + =0
3

II.6 Two vectors of magnitudes A and B make an angle  with each other when placed tail to tail. Prove, by taking components
along two perpendicular axes, that

R = A2 + B 2 + 2 AB cos

gives the magnitude of the resultant R of the two vectors.

II.7. A vector of A of magnitude 10units and another B of 6.0units differ in directions by 60 0 . Find (i) the scalar product of the
two vectors (ii) the magnitude of the vector product

II.8. Given two vectors A = i − 2 j − 4k and B = 3i − j + 5k , find C = AX B

23
PART I: MECHANICS
Mechanics is the part of physics that studies laws of the simplest form of motion of matter, mechanical motion. Mechanical motion is the
variation of mutual position of objects or their parts as entities in space and time. Mechanics consists of three parts: statics, kinematics and
dynamics

Statics: study of laws of equilibrium of bodies

Kinematics investigates characteristics and laws of different kind of mechanical motion of bodies without reference to causes that produce
the motion

Dynamics studies the influence of interaction between bodies on their mechanical motion

Mechanics does not analyze the internal structure of bodies. The simplest model of a body- a point particle: a body whose shape and size can
be ignored in a given problem

The motion of a point particle is very abstract and far from reality but it is a very useful model in mechanics. Even extended bodies can be
investigated using such a model by dividing them into small parts. Each part is analyzed as a point particle and the whole body as a system of
point particles.

Another model used in mechanics is that of an absolutely rigid body: if the distance between any two points of an extended object does not
vary during the motion, the body is called an “absolutely rigid body”. This model is important notably in description of rotational motion.

24
LECTURE III: KINEMATICS OF A POINT PARTICLE
III.1. LECTURE OUTLINE

In this lecture we are going to learn kinematics of a point particle. But before we introduction the main concepts such as displacement,
velocity and acceleration, let us first about reference frame.

Specific objectives

At the end of this lecture, you should be able to:

1. Define a reference frame

2. Define the position vector of a point particle

3. Define the main characteristics of motion of a point particle: displacement, velocity and acceleration

4. Describe particular of motion of a point particle including uniform rectilinear motion, uniformly accelerated rectilinear motion,
free-fall

5. Describe uniform circular motion, UCM

6. Describe projectile motion

III.2. REFERENCE FRAME AND POSITION VECTOR OF A POINT PARTICLE

To analyze motion of a given body, we study it relative to some other bodies. A combination of bodies that are stationary relative to each
other with respect to which motion is being considered and a time piece indicating time form a reference frame. Motion of the same body
relative to different reference frames may have a very different nature.

To describe motion quantitatively we have to associate a coordinate system with the combination of bodies forming the reference frame.
Here, we assume only inertial reference frames i.e. references frames in which Newton’s laws of motion are valid.

The equation for the position vector in the Cartesian system of coordinates is:

r = xi + y j + z k (III.1)

Fig.III.1. Position vector of point M in a system of Cartesian coordinates

25
Vector r is the position vector of point particle M at an initial given moment of time given by t = 0 as shown in Fig.III.1. When the point
particle M moves with respect to the origin O, its coordinates x, y and z and its position vector vary in time. So to study motion of M is to

determine the functions:

x = x(t ) , y = y(t ) and z = z(t ) or r = r (t ) (III.2),

at any given time t

Equations (III.2) are called “kinematic equations of motion” of point particle M.

III.3. DISPLACEMENT

A point particle in motion travels along a certain line. This line as called its “path” or “trajectory”. Depending on the shape of a trajectory,
we can distinguish rectilinear or straight-line motion, circular and curvilinear motion

The length between points A and M measured along the trajectory is called the distance travelled by the particle as shown in Fig.III.2.

Fig.III.2. Displacement and distance travelled by point M. Curve S is the trajectory

The straight line from A to M is called displacement r 12 . The displacement is the increment of the position vector of the particle between
moments of time t1 and t 2 :

r 12 = r (t 2 ) − r (t1 ) (III.3)

The displacement is a vector

The distance travelled is measured along the trajectory; it is a scalar which cannot be negative

Any object in motion obeys the principle of independence of motion that states:

“If a point particle participates in several different motions, the resultant displacement equals the vector or geometrical sum of
displacements each taken separately”.

This is also called ”Principle of superposition of displacements”

26
III.4. VELOCITY AND SPEED

To characterize motion of a particle, we introduce a physical quantity, velocity that characterizes the speed of motion in magnitude and
direction and shows how fast the displacement takes place.

Suppose a particle moving along a curvilinear trajectory MN, so that at moment t it is located at point M and at moment t + t , at point

N. The position vectors of points M and N are r and r +  r respectively; the distance travelled is the length S as we can see in Fig.III.3.

Vector of average velocity v in the interval of time between t and t + t is the ratio of the increment of the position vector, or

displacement  r to the interval of time t

Fig.III.3. Definition of average velocity and velocity of motion

r
v = (III.4)
t

If in the expression (III.4) we go to the limit t → 0 :

r d r
v = lim = (III.5)
t →0 t dt

v is called instantaneous velocity or simply velocity

When time interval t decreases indefinitely, point N tends indefinitely towards M and cord MN revolves around point M. At the limit its
direction coincides with the tangent to the trajectory at point M

→ Vectors d r and v are directed along the tangent to the trajectory in the direction of motion. The limit of the ratio of the arc
S to the cord equals 1 when S → 0

dS
→ d r = dS → v= v =
dt

d r (t )
Mathematically, we will say “velocity is the derivative of the displacement as function of time”: v =
dt

27
v is the rate of change of position vector in time. It is directed along the cord MN

Accordingly, if we integrate v (t ) with respect to time from t to t + dt , we find the length of the path S travelled during the interval of
time t :

t + t
S =  vdt
t
(III.6)

Vector v can be resolved into three components directed along axes of a Cartesian system of coordinates:

v = v x + v y + v z = vx i + v y j + vz k (III.7),

where v x , v y , v z are projections of velocity vector onto coordinate axes

dx dy dz
or v= i+ j+ k,
dt dt dt

dx dy dz
where vx = vy = vz =
dt dt dt
The numerical value or magnitude of velocity:

2 2 2
 dx   dy   dz 
v = v = vx + v y + vz =   +   +  
2 2 2
(III.8)
 dt   dt   dt 

This quantity is called “speed” in English!!!

→ Speed is the numerical value of velocity and it is a scalar and velocity is a vector

→ Speed is the magnitude of velocity.

III.4.1. SOME PARTICULAR CASES

a. Suppose that during motion, direction of vector v does not change in time. Such a motion is called rectilinear, straight-line or
one-dimensional motion

b. If direction of vector v of a particle varies in time, the motion will describe a curvilinear trajectory, and the motion will be called
curvilinear motion, which is a general case

c. If the magnitude or numerical value of velocity does not vary in time, motion is called “uniform”. In this case
t + t
 S = v  dt = vt
t

→ During uniform motion, a particle travels equal distances over equal intervals of time

28
d. Otherwise motion is called “non uniform” which is a general case. We can determine the average speed

S
v = , where S is the displacement made during interval t
t

e. In general v  v : average velocity is different from the average value of velocities

f. If a particle participates in several independent motions, velocity of resultant motion is a vector sum of velocities v i due to separate
motions:

N
v =  vi
i =1

This is the law of superposition of velocities and is the consequence of the law of superposition of displacements

I.5. ACCELERATION

III.5.1.DEFINITION OF ACCELERATION

In general, during motion, the velocity of a particle varies both in magnitude (speed) and in direction. To characterize the rate of change of
velocity, we introduce a physical quantity called “acceleration”.

In Fig.III.3, a particle moves from point M to N and velocity vector varies from v in M to v + v in N. The increment of velocity is v .
If we translate vector v + v to M, v = BD

v
Average acceleration of a non-uniform motion during interval from t to t + t is defined as: a av = a =
t

The direction of a av coincides with the direction of v : a av  v

Acceleration or instantaneous acceleration at moment t is a vector quantity a to which tends average acceleration when t → 0 :

v d v
a = lim a av = lim = (III.9)
t →0 t →0 t dt

d2r
→ a= (III.10)
dt 2

As any vector, acceleration can be resolved into three components:

a = a x + a y + a z = ax i + a y j + az k (III.11),

where a x , a y and a z are projections of vector a onto the respective axes:

29
dv x dv y dv d 2x d2y d 2z
a= i+ j+ z k = 2 i+ 2 j+ 2 k
dt dt dt dt dt dt

and

dv x d 2 x dv y d2y dv z d 2 z
ax = = 2 , ay = = az = = 2 (III.12)
dt dt dt dt 2 dt dt

The numerical value of acceleration:

2 2 2
 d 2x   d 2 y   d 2z 
a = a =  2  +  2  +  2  (III.13)
 dt   dt   dt 

Vector a characterizes the rate of change in time of velocity both in magnitude and in direction. It is possible to resolve vector a into two
components:

- One to characterize rate of change in magnitude only: tangential acceleration, aT

- Another one to characterize rate of change in direction only: normal acceleration, a N

a = a N + aT (III.14),

dv
where aT =  (III.15)
dt

is the tangential component and

d
aN = v n (III.16)
dt

is the normal component of the acceleration of particle at point M.

From expression (III.16) we see that

dv
“The tangent component of acceleration characterizes the rate of change of the magnitude (numerical value) of velocity”: aT =
dt

Consequently:

- In uniform motion : aT = 0

- When aT  0 , motion is said to be accelerated

- When aT  0 motion is said to be decelerated

30
- If aT = const  0 , motion is said to be uniformly accelerated

d
The derivative for a N in (III.17) determines the rate in change of direction of motion. Consequently:
dt

“The normal acceleration characterizes the rate of change in direction of the velocity vector”

Fig.III.4. Tangential and normal components of acceleration are always mutually perpendicular

Radius R is called the radius of curvature (of trajectory at point M)

The centre of the circle is the centre of curvature

v2
aN = (III.17)
R

This projection cannot be negative. Now we see that:

“Vector of normal acceleration is directed along the principal normal towards the centre of the trajectory.

This acceleration is called “centripetal acceleration”

Vectors a T and a N are mutually perpendicular, the magnitude of acceleration is shown in Fig.III.4. The total acceleration of the particle
is:

2
 dv   v 
22
a = a = aT + a N =   +  
2 2
(III.18)
 dt   R 

a
The direction of vector a is given by the angle  : cos  =
a

III.6. PARTICULAR CASES OF MOTION OF A POINT PARTICLE

III.6.1. UNIFORM RECTILINEAR MOTION

For uniform rectilinear motion along the positive direction of OX called also uniform motion in one dimension or one-dimensional

motion: aT = a N = 0 → v = vx i ,
31
where v x = v = const

t
The displacement : x(t ) =  v x dt + x0 = vt + x0 (III.19)
0

x0 is the initial position of particle at t = 0 along the x -axis

III.7.2. UNIFORMLY ACCELERATED RECTILINEAR MOTION

If a N = 0 and aT = const , we have a “uniformly accelerated motion”:

t
dv
aT = → v =  aT dt + v0 = v0 + aT t (III.20)
dt 0

where v0 is the initial velocity at t =0

If the motion is directed along the positive direction of OX :

t
1
x =  vdt + x0 = x0 + v0 t + aT t 2 (III.21)
0
2

aT t 2
The distance traveled s = x − x0 , s = v0 t +
2

Expressions (III.20) and (III.21) can be regarded as the most important expressions for uniformly accelerated rectilinear motion. From these
expressions we can build a table of useful expressions for such a motion:

1. v(t ) = v0 + at

1
2. x − x0 = v0 t + a t2
2

v 2 = v0 + 2a(x − x0 )
2
3.

4. x − x0 =
1
(v0 + v )t
2

Practical Activity III.1

Example III.1

32
m
A car accelerates with constant acceleration from rest to 30 in 10s . It then continues at constant velocity. Find: (a) Its
s
m m
acceleration; (b) how far it travels while speeding up (c) The distance it covers while its velocity changes from 10 to 20
s s
Solution

m
Numerical data: v0 = 0 , v = 30 , t = 10s
s

m m
(a) a =? (b) d =? (c) For v1 = 10 and v 2 = 10 → d12 = ?
s s
Solution: Here we apply directly the equations for uniformly accelerated motion

v 30 − 0 m
(a) a= = =3 2
t 10 s

aT t 2
(b) Equation s = v0 t + gives s =
1 2
at → d=
1
(3)(10)2 = 150m
2 2 2

v 2 − v1
2 2

= v0 + 2a(x − x0 ) gives v 2 = v1 + 2ad d=


2 2 2 2
(c) Equation v → →
2a

d=
(20)2 − (10)2 = 50m
2.(3)

III.6.3. FREE FALL

One particular case of uniformly accelerated rectilinear motion is the free fall with zero initial velocity. For such motion aT = g where g is
called acceleration due to gravity and is constant for all bodies for a given location.

m m
g = 9.806 for Nairobi and normally we take g = 9 .8 for ordinary calculations, by default
s2 s2

Practical Activity III.2

Example III.2

m
A baseball is thrown straight upward with speed of 29.4
(a) How long will it rise? (b) How high will it rise? (c) How long after it
s
m
leaves the hand will it return to the starting point? (d) When will its speed be 16 ?
s

m m
Numerical data: We direct axis upward → a = − g = −9 . 8 , v 0 = 29.4
s2 s

33
v0 29.4
Solution: (a) At the highest point v = 0 . Equation (III.20) → v0 − gt = 0 → t= t= = 3s
g 9.8

1
(b) To find the height reached we substitute t into equation (III.21) → h = − gt 2 + v0 t →
2

Numerical application: h=−


1
(9.8)(3)2 + (9.8)(3) = 44.1m
2
(c) Here we can just reason that the time taken to go up is the same as time taken to go down. But also
mathematically, when the ball gets to ground h=0 → (III.21) →

1 1 2v 0
h = − gt 2 + v 0 t = 0 → − gt + v 0 = 0 → t=
2 2 g

2(29.4 )
Numerical application: t= = 6s
9 .8
III.6.4. UNIFORM CIRCULAR MOTION, UCM

In such a motion, the numerical values of velocity and acceleration remain constant:

v2
v = const → aT = 0 , aN =
R

Acceleration of a body in UCM is called centripetal acceleration and it is at any time and at any point of the trajectory directed towards the
centre of the circle as shown in Fig.III.5

In relation to this motion we can introduce some parameters:

 d
1. Angular velocity of motion :  = lim =
t →0 t dt

In case of UCM,  = const

Fig.III.5. Velocity and acceleration in uniform circular motion

34
The SI unit of angular velocity:

  =   = rad
t  s
(III.22)

2. Linear velocity v = R

3. Period of revolution T : time for the body to complete one revolution or cycle around the circle: for  = 2 :

t = T for an angular displacement of  = 2

2 2
= or T= (III.23)
T 

4. The number of revolutions or turns in unit of time,  is called frequency of motion or number of revolutions
per second

1 
= = (III.24)
T 2
The SI unit for  is

  = 1rev  1Hertz = 1Hz (III.25)


s

Engineers like measuring the speed of rotation of machines in RPM , revolutions per minute. Obviously

1rev 1rev 1 rev


1RPM = = = (III.26)
min 60 s 60 s

Practical Activity III.3

Example III.3

A car travelling at constant speed takes 80s to complete one lap of a circular track of length 2.4km . What is the centripetal
acceleration?

Solution

Numerical data: T = 80s ,  = 2R = 2.4km = 2.4 X 10 3 m , acent = ?

v2
Solution: Centripetal acceleration is given by: a cent = where v = R (1)
R


and R= (2).
2

2 2
Here = (3) is the angular velocity. Combining (1), (2) and (3), we get: a cent =
T T2

35
2
Working equation: a cent =
T2

2 2.4 X 10 3 m
Numerical application: acent = = 2.356
(80) 2
s2

m
Answer: a cent = 2.356
s2

III.7. PROJECTILE MOTION

This is a case of two-dimensional motion. A particle moves in a vertical plane with initial velocity v 0 but its acceleration is always the free-
fall acceleration g . Such a particle is called a projectile. Fig.III.6 shows the direction of velocity and its components at different points of
trajectory

In projectile motion, the horizontal motion and vertical motion are independent of each other, neither motion affects the other. We can apply
principle of independence of motion, here. So we analyze them independently of one another

The initial velocity of the projectile is given by (see Fig.III.6):

v 0 = v0 x i + v0 y j (III.27),

where

vox = v0 cos  0 voy = v0 sin  0 , (III.28)

where  0 is called the angle of elevation or launch angle

Horizontal motion

The horizontal motion is uniform rectilinear with velocity: vox = v0 cos  0 →

x = v0 x = (v0 cos  0 )t (III.29)

Vertical motion

For vertical motion, the acceleration is constant and equal −g because we have directed the y axis upwards

36
Fig.III.6. Velocity and its components for projectile motion

From (III.21):

gt = (v0 sin  0 )t − gt 2
1 2 1
y = v0 y t − (III.30)
2 2

v y = v0 sin  0 − gt (III.31)

The vertical velocity component behaves just like for a ball thrown vertically upwards. It is directed upward initially and its magnitude
steadily decreases to zero, which marks the maximum height of the path

The vertical velocity component then reverses direction and its magnitude increases with time till the ball reaches the ground

Equation of the path or trajectory

We can notice that the trajectory lies in a vertical plane. If we eliminate t in equation (III.29-III.31) and rearrange the equation, we get:

x2
y = (tan  0 )x −
1
g (III.32)
2 (v0 cos  0 )2

This equation can be written as y = ax + bx 2 : a parabola → The trajectory of a projectile is parabolic

Horizontal range

The horizontal range R of a projectile is the horizontal distance the projectile travels when it returns to its initial launch height

Let’s put x = R in (III.29) and y = 0 in (III.30):

R = (v0 cos  0 )t 0 = (v0 sin  0 )t −


1 2
gt
2
2
v0
If we eliminate t → R=2 sin  0 . cos  0 →
g
2
v0
R= sin 2 0 (III.33)
g
37
→ The horizontal range R is maximum for a launch angle of 45 0 for a given value of initial velocity v0

III.8. GRAPHICAL REPRESENTATION OF MOTION

Motion can represented graphically and graphical representation can be of great help in solving some problems of kinematics

III.8.1. UNIFORM RECTILINEAR MOTION

For such a motion acceleration is given by aT = a N = 0 . Therefore the velocity is: v = v x i = const

Suppose vx = v = v  0 .

The distance traveled during interval t :

t0

x(t ) =  vdt = vt0


0

This is the area of a rectangle with horizontal side 0 → t o and vertical side 0 → v as shown in Fig.III.7 on a (t, v) plot

Fig.III.7. Displacement in uniform rectilinear motion on a (t, v) plot


The graph for velocity is a horizontal line parallel to x -axis at level v (Fig.III.7)

III.8.2. UNIFORMLY ACCELERATED RECTILINEAR MOTION

t
For such a motion aT = const , a N = 0 . Therefore v(t ) =  adt = at + v0 ,
0

t t
x(t ) =  v(t )dt =  (at + v0 )dt =
1 2
at + v0 t + x0
0 0
2

The three graphs for quantities x(t ) , v(t ) , a(t ) are given in Fig.III.8

38
Fig.III.8. Displacement, velocity and acceleration in uniformly accelerated motion

The distance travelled can be geometrically calculated from the graph v = v(t ), according to Fig.III.9

Fig.III.9. Graph v = v(t )uniformly accelerated motion

= v(t ): A = A1 + A2 , where A1 = v0 t and A2 = t.(at )


1
The distance travelled is equal to the area of the figure below the graph v
2
1
→ A = v0 t + at 2
2
This expression is exactly the same as (III.21)

39
III.8.3. GENERAL CASE

For a general case, the distance travelled can be found by integrating function v = v(t ) in the limits of the duration of the motion as to
calculate the area of the figure under the graph

t2
d =  v(t )dt (III.35),
t1

as shown in Fig.III.10

Fig.III.10. Calculating the distance travelled by computing the area of the figure under the graph v = v(t )

Summary
Kinematics is the study of motion with out reference to its causes. The three kinematic variables of a point particle are:

(i) Displacement : r (t ) , Velocity: v = v(t ) , Acceleration: a = a(t )

The displacement is the change in position vector of the particle in time:  r = r (t ) − r 0 ,

where r0 is the initial position of the particle at t = 0

r
The average velocity for a time interval t : v AV =
t

v(t ) =
dr
The instantaneous velocity or simply velocity:
dt

is the rate of change of position with respect to time. It is given by the slope of the tangent to the graph r (t ) and is always directed along the
tangent to the trajectory at a given point.

For a three-dimensional motion we have to take in account the three components of all variables of motion along the corresponding axes:
This is the principle of independence of motion.

Therefore the magnitude of velocity is given by:

40
2 2 2
 dx   dy   dz 
v = v = vx + v y + vz =   +   +  
2 2 2
;
 dt   dt   dt 

dq
where vq = is the velocity component along q axis
dt

dv d 2 r
Instantaneous acceleration or simply acceleration : a= = 2
dt dt
In curvilinear motion acceleration has two components:

(i) Tangential acceleration a is responsible of the variation of the magnitude of velocity and is directed along the tangent to the
dv
trajectory at the point: a = 
dt

(ii) Normal acceleration, a N , responsible of variation of direction of velocity and directed toward the centre of curvature at the point:
v2
an =
R

2
 dv   v 
2 2
a = a = a + a n =   +  
2 2
Therefore, the total acceleration of the particle is:
 dt   R 

Some particular cases:

1. No change in direction: one-dimensional, or rectilinear motion : an = 0 , a  a

(i) Rectilinear uniformly accelerated motion. a = const

v(t ) = v0 + at , x(t ) = x0 + v0 t +
1 2
The kinematic variables are given by: a = const , at
2

m
For the free fall a = g and always directed downwards. By default g = 9.8 . g is called the acceleration due to gravity or
s2
acceleration of free fall

(ii) For the general case: v(t ) =  a(t )dt and x(t ) =  v(t )dt

2. Uniform circular motion: a = 0 , a n = const and at any point of the circle directed toward the centre; this is centripetal

v2
acceleration whose magnitude is a cent =
R

41
x2
y = (tan  0 )x −
1
3. Projectile motion: (i) The equation of the trajectory: g
2 (v0 cos  0 )2
2
v0
(ii) The range: R= sin 2 0
g

Self-test III

III.1. Describe a physical situation for example with a ball or a car, for each of the following cases. State whether the object is speeding
up or slowing down.

(i) a = 0, v  0 (ii) v = 0 , a  0 (iii) v  0 , a  0 (iv) v  0, a  0

III.2. A journey consists of two segments, each at constant velocity. Under what condition would the average velocity for the whole trip
equal the average of the two velocities?

III.3. An object thrown vertically up is instantaneously at rest at the highest point. What is its acceleration at that point?

III.4. Ball A is thrown vertically up whereas ball B is thrown down with the same initial speed, from the same rooftop. Compare the
speeds with which they land in the absence of air resistance

III.5. A ball is thrown vertically up and then falls back to the ground. Which of the graphs in Fig.III.11 best represents the variation of its
velocity with time?

III.6. Can a particle move at constant speed and yet be accelerating? If so, give an example

III.7. True or false: When a particle moves in uniform circular motion its acceleration is constant

Fig. III.11. Question III.5


42
Practical activity III.4: Tutorial
1. An automobile travels on a straight road for 36km at 36km/ h . It then continues in the same direction or another
36km at 72km/ h . (a) What is the average velocity of the car during this 72km trip(b) What is the average speed (c) Graph
x versus t and indicate how the average velocity is found on the graph

2. A car traveling 54.0km/ h is 24m from a barrier when the driver slams on the brakes. The car hits the barrier 2.00s later. (a)
What is the car’s constant deceleration? (b) How fast is the car traveling at impact?

3. At a construction site a pipe wrench struck the ground with speed of 29.4m/ s (a) from what height was it inadvertently
dropped? (b) How long was falling? (c) Sketch graphs of y , v and a versus t for the wrench

0
4. A soccer ball is kicked from the ground with an initial speed of 19.6m / s at an angle of 45 . A player 60m away in the
direction of the kick starts running to meet the ball at that instant. What must be his average speed if he is to meet the ball just
before it hits the ground? Neglect air resistance.

5. The fast French train known as the TGV has a scheduled average speed of 216km/ h (a) I the train goes around a curve at that
speed and the magnitude o the acceleration experienced by the passengers is to be limited to 0.050 g , what is the smallest radius
of curvature for the track that can be tolerated? (b) At what speed must the train go around a curve with a 1.00km radius to be at
the acceleration limit

Practical activity III.5: Assignment

1. 18km/ h . Then during the half of the remaining


A cyclist rides from city A to city B. The first half of the way, he travels at a speed of
time, he travels at a speed of 9km/ h and during the other half of the time at a speed of 4.5km/ h . What is the average speed for the
whole trip?

3. A Jumbo jet must reach a speed of 360km/ h on the runway for takeoff. What is the least constant acceleration needed or takeoff
from a 1.8km runway?

4. A hoodlum throws a stone vertically downward with an initial speed of 12.0m / s from the roof of a building, 30.0m above the
ground. (a) How long does it take the stone to reach the ground? (b) What is the speed of the stone?

5. 25m/ s . Within what two elevation angles must he kick the ball to score a goal from a
A striker can give the ball an initial speed of
point 50m in front of the goalposts whose horizontal bar is 3.44m above the ground?

6. A boy whirls a stone in a horizontal circle of a radius 1.5m 2.0m above level ground. The string breaks and the stone
at height
flies off horizontally and strikes the ground after travelling a horizontal distance of 10m . What is the magnitude of the centripetal
acceleration of the stone while in circular motion?

43
LECTURE IV: PARTICLE DYNAMICS
IV.1. LECTURE OUTLINE

In this lecture we are going to study the fundamental laws of dynamics of a point particle. Classical mechanics or Newtonian mechanics is
based on three laws of dynamics that were formulated by Sir Isaac Newton in 1687

The Newtonian laws are based on experimental facts

Newtonian mechanics achieved such a success in the two centuries that many physicists were convinced in its omnipotence

But the development of relativistic mechanics - mechanics of bodies with velocities almost equal to velocity of light v  c and quantum
mechanics, mechanics of elementary particles- atoms, electrons… have shown that Newtonian mechanics is correct only for bodies with
velocity much less than the velocity of light and of large mass compared with the mass of atoms.

IV.2. SPECIFIC OBJECTIVES

At the end of this lecture, you should be able to:

1. Define linear momentum of a particle in motion and formulate the law of conservation of linear momentum
2. Formulate the principle of inertia

3. Clearly formulate the fundamental law of dynamics of a particle also known as the Newton’s second law of motion.

4. From that fundamental law to clearly define the main dynamic characteristics of motion of a point particle such mass, force…

5. Formulate clearly the Newton’s third law

6. Describe the types of forces encountered in mechanics

7. Apply the laws of dynamics of a particle in solving problems of motion of a particle

IV.3. NEWTON’S FIRST LAW: PRINCIPLE OF INERTIA

The Newton’s first law says: “Any object remains in its state of rest or of uniform rectilinear motion unless it is compelled by external
forces to change that state”

The action of other bodies on a given body causes its velocity to change: it imparts acceleration on the body.

Any object resists attempts to change its state of motion (or rest). This property of all bodies is called “inertia”

It is characterized quantitatively by a physical quantity called the “mass” of a given body.

The product of the mass of a body and its velocity is called its “(linear) momentum”:

p = mv (IV.1)

If we are dealing with an extended body, we can imagine the body as consisting of small particles of mass mi , i = 1,2,3... determine the

momentum mi v i of each and add those momenta as vectors:

44
N
p =  mi v i (IV.2)
i =1

A group of assorted particles m1 , m2 … m N form a “system of particles”.

A system of particles m1 , m2 … m N that interact with one another and with no other body out of the system is called an “isolated system of
particles“

IV.5. LAW OF CONSERVATION OF MOMENTUM

The law of conservation of momentum says: “The total momentum of an isolated system of particles remains constant”:

N
p =  p i = const (IV.3)
i =1

IV.6. NEWTON’S SECOND LAW OF MOTION

Newton’s second law of motion also known as the fundamental principle of dynamics of a point particle states:

“The rate of change of momentum of a body with respect to time equals the resultant or net force  F acting on the body”:
dp
= F (IV.4)
dt
Equation (IV.4) is called the fundamental equation of dynamics of a point particle

d p d mv
= =v
dm ( )
+m
dv
(IV.5)
dt dt dt dt

dm
If m = const → v =0 →
dt

dv
m = ma =  F (IV.6)
dt
The product of the mass of the body and its acceleration is numerically equal to the net force acting on the body - fundamental law of
dynamics

In this context, a” force “can be defined as a “physical quantity that characterizes the action of a body on another”. It is a quantitative
measure of interaction between two particles.

A force is a vector characterized by a magnitude, a point of application and line of action (direction)

In the same context we can define mass of a body:

Suppose an object P. If we apply a net force F1 , it gets an acceleration a1 ; if we apply on the same object a net force F2 , it gets an

F1 F2
acceleration a 2 and so on.. If we calculate the quotients , … we notice that they are equal:
a1 a2
45
F1 F2 F
= = ... = N = const (IV.7)
a1 a2 aN

The constant given by (IV.7) does depend neither on force nor on acceleration. It is a propriety of the body and it is called its “mass”

The mass is a quantitative measure of inertia of a body because it reflects its resistance of getting acceleration when acted upon by a force

For a given force F , the greater the mass of a body the less acceleration it will get

IV.7. UNIT OF FORCE

If you remember

v=
x
→ v = x = m
t  s
1. Unit of velocity:
t

m
a=
v
→ a = v = s = m.s −2
t  s
2. Unit of acceleration:
t

Consequently,

3. Unit of force: F = ma → F  = m. a = kg.m.s −2

This unit 1kg.m.s −2 has been given a name of 1Newton :

1kg.m.s −2  1Newton  1N (IV.8)

m
We can define 1 Newton as “Such a force which causes an acceleration of 1 when it acts upon a body of 1kg of mass”
s2

Practical Activity IV.1

Example IV.1

An airplane has a mass of m A = 3.1X 10 4 kg and takes off under the influence of a constant net force FA = 3.7 X 10 4 N .

What is the net force that acts on the plane’s pilot of mP = 78kg ?

Solution

Numerical data in SI: Mass of airplane M = 3.1X 10 4 kg

Force acting on airplane, FA = 3.7 X 10 4 N

Mass of the pilot: mP = 78kg

==========================================

46
Force acting on pilot FP = ?

Solution

According to Newton’s second law, the acceleration is a=


 F . Since the pilot and the plane have the same acceleration, we can
m
write: a P = a A where a P is the acceleration of the pilot and a A is the acceleration of the airplane

FP F FA mP
aP = a A → = A → FP = - working equation
mP m A mA

3.7 X 10 4 NX 78kg
Numerical application: FP = = 94 N
3.1X 10 4 kg

Answer: Force acting on the pilot FP = 94N

Example IV.2

m
A 1600kg - car is traveling with a speed of 15.0 . What is the magnitude of the horizontal net force that is required to bring the
s
car to a halt in a distance of 50.0m ?

m
Numerical data in SI: m = 1580kg , v 0 = 15.0 , v1 = 0 , d = 50.0m F =?
s
Solution

v1 − v 0
2 2 2
v
The acceleration required is: a= =− 0
2d 2d
2
v0
Newton’s second law:  F = ma → F = − m. - working equation
2d

 F = −1600kg.
(15)
2
m2
= 3600 N
(2 X 50)m s 2
Numerical application:

Answer: F = 3600N

Example IV.3

A boat has a mass of m = 6800kg . Its engines generate a drive force of 4100 N , due West, while the wind exerts a force of
800N , due East, and the water exerts a resistive force of 1200N due East. What is the magnitude and direction of the boat’s
acceleration?

Numerical data in SI: m = 6800kg , FM = 4100N , FW = 800 N , FR = 1200 N a=?


47
Solution

Newton’s second law of motion gives the relationship between the net force F and the acceleration a that it causes for an object of
mass m . The net force is the vector sum of all the external forces that act on the object. Here the external forces are the drive force, the force
due to the wind, and the resistive force of the water.
We choose the direction of the drive force (due West) as the positive direction. Solving Newton’s second law for the acceleration

gives  F = ma . The problem is one-dimensional along the line East-West: Forces oriented west are positive and those directed east are
negative.

F FM − FW − FR
Newton’s second law: F = ma → a= = working equation
m m

4100 − 800 − 1200 m


Numerical application: a= = +0.31 2
6800 s
Observation: Acceleration is positive → the boat heads West!!!

IV.8. NEWTON’S THIRD LAW OF MOTION

The Newton’s third law states: “If a body A acts on a body B with a force F BA , then automatically body B acts on body A with a force
F AB such that

F BA = −F AB (IV.9)

The forces exerted by interaction of bodies on each other are equal in magnitude and opposite in direction.

These bodies are then called ”third-law pair”

This law is also called “principle of action and reaction”

IV.9. TYPES OF FORCES

Four types of interactions are known in modern physics:

- Gravitational: This is interaction due to universal gravitation (because of the mass of objects) this is a
long-range interaction
- Electromagnetic: This interaction is achieved via electric fields between charged particles
- Strong or nuclear: This interaction ensures the binding of particles in atomic nucleus. This is short-range
interaction
- Weak interaction: This interaction is responsible for many processes at elementary particle level

In classical mechanics, we deal mainly with gravitational, elastic and friction forces

Gravitational and electromagnetic interactions are fundamental: they cannot be reduced to other simpler interactions

Elastic and friction forces are secondary because they are due to molecular interactions which are electromagnetic by nature

48
IV.5.1. LAWS OF FUNDAMENTAL FORCES

m1m2
Gravitational: F =G e12 - Newton’s law of universal gravitation
r2

q1q 2
Electromagnetic: F = −k e12 - Coulomb’s law of electrostatics
r2
IV.10. ELASTIC FORCES

Any real object deforms or changes its dimensions and shape under the action of forces applied on it. If the object regains its initial
dimensions and shape after the action of forces stops, then the deformation or strain is called “elastic”

If the body does not regain its initial characteristics after the interruption of external action, then the deformation is called “plastic”

Elastic deformations are observed when the force when the force producing them does not exceed a definite limit called elastic limit for each
concrete object

Let us take a spring of length x 0 in its undeformed or equilibrium state as shown in Fig.IV.1

Fig.IV.1. Illustration of external force acting on an elastic object

A spring is used as a one-dimensional model for an absolutely elastic body. We apply an external force Fext on end while the other end is

solidly attached to a support. Under that force, the spring will stretch over a certain distance x , after that the equilibrium will set in. At
equilibrium the external force Fext will be balanced by an elastic force set up in the spring as a result of its deformation

Definition: Elastic force is a force which develops in a body to oppose or resist deformation

Experiments show that for small deformations (below elastic limit), the elongation x is proportional to the external force Fext : x  Fext

Elastic force is equal and opposite the external force (third-law pair):

F el = −kx (IV.9)

Expression (IV.9) is called “Hooke*’s law”

→ An elastic force is always directed in the opposite direction of the elongation.

49
This means that the elastic force is always directed towards the position of equilibrium and is called a restoring force: it tends to restore the
body in its position of equilibrium

*Robert Hooke FRS (28 July 1635 – 3 March 1703) was an English natural philosopher, architect and polymath.

IV.11. FRICTION FORCES

Forces of friction appear when bodies in contact or their parts move or attempt to move relative to one another.

Friction occurring in the relative movement of two contacting bodies is called ”external”, friction between parts of the same body, for
example in a fluid, is called “internal”

Friction between the surfaces of two solids in the absence of any intermediate layer (a lubricant) is called dry friction

Friction between a solid and a fluid, also between layers of a fluid is called viscous or liquid friction.

There are two kinds of dry friction: sliding and rolling friction

IV.11.1. FORCE OF DRY FRICTION

A force of friction appears not only when one surface slides or rolls over another one but also when there is an attempt to set up such a
sliding or rolling motion. This force is called” force of static friction” as shown in Fig.IV.2.

Fig.IV.2. Static friction

Because there is no acceleration: F X = 0 →

Fst = Fext (IV.10)

“Force of static friction is always equal the external force”

When there is relative motion between the two surfaces, we have dynamic or kinetic friction.

Force of dynamic friction is proportion to the force of reaction of the surface on the object:

F fr = − k fr N (IV.11)

k fr - Coefficient of kinetic friction,

N = FN - Normal force with which the support acts on the body called also reaction force

50
The sign – always present in the expression for friction force means that friction force is always opposite to motion

IV.11.2. VISCOUS FRICTION AND RESISTANCE OF THE MEDIUM

Suppose a solid body moving in a fluid. The motion is subjected to a force called force of resistance of the medium. Experiments show that
the resistance force depends on the velocity of the body.

At low velocities:

F fr = −k1 v 1 (IV.12)

At higher velocities:

F fr = −k 2 v 2 e v (IV.13)

Coefficients k1 and k 2 depend on the shape, the size, the state of the surface and on the propriety of the fluid called viscosity. They can only
be determined experimentally

IV.12. FORCE OF GRAVITY AND WEIGHT

Force of attraction toward the Earth causes all bodies to fall down with the same acceleration g relative to the Earth’s surface

Any particle of mass m is acted upon by a force

F g = mg (IV.14)

Fg is called force due to gravity or force of gravity or gravitational force

When a body is at rest relative to the Earth surface, the force Fg is balanced by a reaction force Fr of the suspension or support which
prevents the body from falling:

Fr = − F g (IV.15)

According to Newton’s third law, the body acts on the suspension or support with a force W equal to Fg :

W = F g = mg (IV.16)

Force W with which a body acts on its support is called the weight of the body

This force equals m g only when the body and its support are stationary relative to the Earth or move uniformly and in straight line (inertial
reference frame)

Anyway Fg and W are different forces because Fg is applied to the body and W is applied to the support. They can eventually have the
same magnitude and direction

51
If the body and the support move with acceleration W will not be equal to m g

Let a suspension in the form of a spring fastened with a frame move together with a body with acceleration

The equation of motion for the body:

F = F g + F r = ma (IV.17)

where Fr is the reaction force of the suspension: force with which the spring acts on the body

The body acts on the springs with a force equal to − Fr which the weight of the body by definition

1. a = 0, F = 0 → Fg = − Fr = W → W = mg

2. a  0,  F =F g + F r = ma → Fr = ma − Fg → W = −Fr

W = Fg − ma = m(g − a ) (IV.18)

Suppose that the body and suspension are moving in vertical direction:

W = m(g  a ) (IV.19)

a  0 , if directed upwards

a  0 , if directed downwards

The magnitude of the weight W may be either greater or smaller than the force of gravity Fg

In the case of free fall of the frame with the suspension, acceleration a = g will make the force with which the body acts on the support or
suspension vanish in (IV.18). This is called “state of weightlessness”

Often the force of gravity Fg is confused with the weight W of a body, but:

1. The two forces are applied to different bodies : Fg is applied to the body itself, W is applied to the support which prevents the
body from falling in the Earth gravitational field

2. Fg always equals mg . W depends on the acceleration with which the body and the support are moving in the vertical direction

Practical activity IV.2

Example IV.4

52
Suppose, as shown in Fig.IV.3 that a 70kg -box is pulled by a force by a 400 N force at an angle of 30 0 to the horizontal. The
coefficient of sliding friction is 0.50 . Find the acceleration of the box.

Fig.IV.3. Example IV.4

Solution

Numerical data in SI: m = 70kg , F = 400N ,  = 30 0 , k fr = 0.5 ,


a=?

Reasoning

The problem is one-dimensional; we direct the x -axis along the direction of motion and apply Newton’s second law: F = ma .

The force acting on the box along x -axis are: FX along x -axis: positive and the friction force F fr opposite x -axis as usual: negative,
where FX = F cos and F fr = k fr N ( N is represented by Y on the diagram).

As there is no acceleration along y -axis the sum of all forces along y -axis is zero as shown in the diagram: mg − F sin  − N = 0 →
N = mg − F sin  and F fr = − k fr N .

Newton’s law gives:

F cos  − k fr (mg − F sin  ) = ma →

F cos  − k fr (mg − F sin  )


a= Working equation
m

400 X 0.866 − 0.5(70 X 9.8 − 400 X 0.5) m


Numerical application: a= = 1.47 2
70 s

m
Answer: a = 1.47
s2
Example IV.5

53
Fig.IV.4 shows a body of mass 12kg placed on an incline which forms an angle of 30 0 with the horizontal. To the body is attached
a light unstretchable string which via a light pulley is attached to a hanging body of 20kg . Determine the acceleration of the bodies
and the tension of the string if the coefficient of friction is 0.1 . Mass of the pulley and the friction on it can be ignored.

Fig.IV.4 Example IV.5

m
Numerical data in SI: m1 = 12kg ,  = 30 0 , m2 = 20kg , k fr = 0.1 , g = 9.8 ,
s2
a=?
Reasoning and solution

The basic fact is that the acceleration of the two bodies is the same: a1 = a2 = a . Body m1 is acted on by the Earth with

force Fg 1 = m1 g and by the incline with force N = m1 g cos  .

The resultant of the two forces is F1 = m1 g sin  as shown in the figure.

When the body moves along the incline a friction force acts on it. This force is proportional to force Q : F fr = k fr Q . Force of friction is
always directed opposite motion or eventual motion.

In this problem force Q = m1 g cos  . Actually this force is the weight of the body on the incline!!!!

Newton’s third law: N = −Q

For body m1 , equations are: m1 a = T − F1 − F fr → m1 a = T − m1 g sin  − k fr m1 g cos  (1)

For body m2 : m2 a = m2 g − T (2)

We have two equations from which we have to determine two quantities a and T . We find them as:

m2 g − m1 g sin  − k fr m1 g cos 
a= : Working equation (3)
m1 + m2

54
m2 g + m1 g sin  + k fr m1 g cos 
T = m2 Working equation (4)
m1 + m2

m
Numerical application: a4 , T  121N
s2

IV.13. UNIFORM CIRCULAR MOTION

When a body moves in a circle at constant speed v , it is said to be in uniform circular motion, UCM. Such a motion is very important in
cases such as planetary motion, machinery…

In Lecture III, we have seen that such a body has a centripetal acceleration directed toward the centre of the circle and of constant
magnitude

v2
a= (IV.20)
R

If there is acceleration there must be a force. This force is called “centripetal force”

Definition: “A centripetal force is the resultant of all forces acting on a body in UCM”

A centripetal force accelerates a body by changing the direction of the body’s velocity without changing the body’s speed and it is equal to

v2
Fc = m (IV.21)
R

Because the speed v is constant, so are also the magnitudes of the acceleration and the force. However, the directions of the centripetal
acceleration and force are not constant they vary continuously so as to always point toward the centre of the circle.

Centripetal force is not a type of force such as gravitational force or friction;; it is a role played by any force that makes an object move in
UCM. That force can be the gravitational force, friction force, electrostatic force….or a combination of forces.

Practical Activity IV.3

Example IV.6

A small coin is placed at the rim of a turntable of radius 15cm which rotates at 30RPM . Find the minimum coefficient of friction
for the coin to stay on

Numerical data in SI: R = 15cm = 0.15m

rev rev 1 rev rad


f = 30 RPM = 30 = 30 X = 0. 5 ;  = 2f → = 2f = 
min min s s s
60
min
Solution

The situation is illustrated in Fig.IV.5, where in side view we see the three forces acting on the coin

55
Fig.IV.5 Example IV.6

The centripetal acceleration is directed toward the centre so we choose the + x axis in that direction. The necessary centripetal force is
provided by the force of friction f . Newton’s second law gives:

ma =  F = N + f + m g

mv 2
Its components are: FX = f =
R

As there is no acceleration in the vertical direction → FY = N − mg = 0

mv 2 v2
As f = k fr N = k fr mg → k fr mg = → k fr = working equation
R R

Numerical application: k fr = 0.15

This is the minimum friction coefficient to maintain the coin on the turntable! Notice that the mass does intervene in the expression!!!!

Summary

1. Linear momentum of a particle is: p = mv

2. Newton’s first law of motion also known as the principle of inertia:

“Any object remains in its state of rest or of uniform rectilinear motion unless it is compelled by external forces to change that state” or

“The linear momentum of an isolated system cannot change”:  F ext = 0  p = const

3. Newton’s second law of motion also known as the fundamental principle of motion of a particle:

“The rate of change of momentum of a body with respect to time equals the resultant or net force  F acting on the body”:
dp
=  Fext or ma = F ext
dt

56
The mass of a body is the quantitative measure of inertia of the body

A force is a quantitative measure of interactions between two particles

1m
The SI unit of force is: 1kg. = 1Newton = 1N
s2
4. Newton’s third law of motion also known as the principle of action and reaction

“If a body A acts on a body B with a force F BA , then automatically body B acts on body A with a force F AB such that:

F BA = −F AB

In mechanics we consider three types of forces: gravitational, elastic and friction forces

(i) Gravitational force: F g = mg :

(ii) Elastic force: F el = −kx (Hooke’s law)

(iii) Friction force: F fr = −k fr N

mv 2
When a particle is in uniform circular motion the net force acting on it equals the centripetal force: F cent =
R

Questions IV

IV.1. Answer the following true/false statements with a brief explanation:

(i) A nonzero net force acting on a body always changes its speed

(ii) A person in a free-falling elevator is weightless

(iii) While pushing a crate along the floor, you must exert greater force on it than it does on you

IV.2. Consider the statement: “in a tug-of-war, the two teams exert equal force on each other”. Do you accept this? Explain why one
team wins.

IV.3. A ball thrown vertically upwards momentarily comes to rest at its highest point. Is it in equilibrium at this instant? Explain why or
why not

IV.4 A small car and a huge truck collide. At any instant, which experiences (i) the greater force (ii) the greater acceleration

IV.5 An astronaut in circular orbit around the Earth feels weightless. Why is this?

IV.6. A coin is at the rim of a rotating turntable.When the rate of rotation reaches a certain value, why does the coin fly off

IV.7. When a plane moves in a vertical circle, at what point is the danger of pilot blackout greatest?

IV.8. Why does not a satellite in stable orbit fall back to Earth? Ignore air friction

57
Practical Activity IV.4

Exercises IV

1. A horizontal cable pulls a 200 − kg cart along a horizontal track. The tension in the cable is 500N , starting from rest (a) How
m
long will it take the cart to reach a speed of 8 ? (b) How far will it have gone?
s

m
2. A 2kg particle is subject to two forces that produce a resultant acceleration a = 4i − 3 j , If
s2
F1 = −i + 2 j + 3k , N , find F2

3. Two blocks of masses m1 = 5kg and m2 = 6kg are on either side of the wedge in Fig.IV.6. Find their acceleration and
the tension in the rope. Ignore friction and the pulley

Fig.IV.6. Exercise IV. 3

4. A 882 − N man stands on the floor of an elevator. The scale records the force it exerts on whatever is on it. What is the scale
m m m
reading if the elevator has an acceleration of (a) 1.8 2 up (b) 1.8 2 down (c) 9.8 2 down?
s s s

km
5. A downhill skier starts from rest down a 40 0 slope for which k fr = 0.1 (i) How long does it take to reach 90 (ii) What
h
is the distance covered?

6. A certain car of mass m has a maximum frictional force of 0.7mg between it and the pavement as it rounds a curve on a flat
road ( = 0.7) . How fast can the car be moving if it is to successfully negotiate a curve of 15m radius?

7. A car moves in uniform acceleration motion around a circle of radius R = 25m. From rest, it travels a distance of 50m in 10s .
Find the centripetal acceleration and total acceleration of the car at end of journey

Practical Activity IV.5

Problems IV

58
km
1. An 11,000kg wagon is moving at 18 . What must be the braking force for it to stop within in a distance of 250m ?
h

2. The breaking strength of a steel cable is 2.5kN . With what maximum acceleration a load of 0.2T can be lifted using the cable
without breaking it?

3. Two forces F 1 = i + 2 j, N and F2 which 4 N directed at 37 0 measured from the x axis toward the y axis, act on a

200 g particle. What is the acceleration?

m
4. A car of mass 1.2T starts moving with acceleration a = 1.04 What is the force of traction of the car if the friction coefficient
s2
is  s = 0.2 ?

5. In Fig.IV.6, two blocks are connected over a pulley. The mass of block A is 10kg and the coefficient of kinetic friction between
A and incline is 0.20 . Angle  = 40 0 . Block A slides down the incline at constant speed. What is the mass of block B?

Fig.IV.6 Problem 4.

km
6. An airplane is flying in the vertical plane along a circular arc with a speed of 900 (Fig.IV.7). Calculate the radius of
h
curvature if at the lowest point B of the trajectory, the acceleration of the aircraft equals 10 g

Fig.IV.7 Problem 5

59
LECTURE V: WORK ENERGY POWER

V.1. LECTURE OUTLINE

Definitely, physics can be defined as the study of matter and energy, what makes energy one of the most important concepts in physics. Here,
we will be interested in mechanical energy and work is one of forms of mechanical energy when a force displaces its point of application a
certain distance. Another form of mechanical energy is kinetic energy possessed by particles in motion and the other form is mechanical
potential energy that depends on the mutual position of particles in a certain system. Energy can transform from one form to another and
when there are no dissipative forces in the system, the total amount of energy remains constant. This is the famous principle of conservation
of total mechanical energy which is a particular case of the general principle of conservation of energy. Power is simply the rate on which
work is done or energy is transformed with respect to time.

Specific objectives

At the end of this lecture, you should be able to:

7. Define work done by a force

8. Define developed by a force

9. Define and relate work and kinetic energy

10. Define and differentiate gravitational and elastic potential energy

11. Define and differentiate conservative and dissipative systems

12. State and apply the principle of conservation of total mechanical energy

V.2. ENERGY AND WORK

Energy is the universal measure of different forms of motion that accounts quantitatively and qualitatively for transformations of one form
of motion into another.

Motion is a form of existence of matter. Thus any object possesses energy or stored energy. This energy can appear in different forms:
mechanical, chemical, electromagnetic…

In mechanics we talk mainly about mechanical energy and its transformations

Change in motion of a body and its energy is due to interaction of the body with other bodies. To characterize this change in mechanics the
notion of work is introduced

V.2.1. WORK

Suppose a force F = const applied to the body so that it moves in a straight line. The work done by the force F when the body travels a

distance S is by definition:

W = F.S = FS cos  (V.1)

SI Unit for work

1Newton acts on a distance of 1metre in the same direction as the direction of the body, then it performs work equal to
If a force of
1NewtonX1metre or 1Joule . Thus the SI unit for work is Joule . This unit was named in the honour of James Prescott Joule*

W  = F . S cos = 1N.1m  1Joule  1J (V.2)

60
*James Prescott Joule FRS (24 December 1818 – 11 October 1889) was an English physicist and brewer, born in Salford, Lancashire.
Joule studied the nature of heat, and discovered its relationship to mechanical work . This led to the theory of conservation of energy, which
led to the development of the first law of thermodynamics

Practical Activity V.1

Example V.1

A skier of mass m = 40kg is given a displacement of 20m along a slope inclined at  = 15 0 to the horizontal. The tension in the
towrope is T = 250N and acts at an angle  = 30 0 (See Fig.IV.1) Given that k fr = 0.1 , find the work done by each force and
the net work on the skier

Fig. V.1 Example V.1

Solution

The four forces acting on the skier are shown in the figure.

From the condition F y =0 → N = mg cos  − T sin  = 379 N − 125 N = 254 N


→ F fr = k fr N = 25.4 N .

From the figure the work done by each force:

(i) WT = T .s = Ts cos 30 0 = +4330 J (ii) W fr = F fr s = − F fr .s = −508 J

(iii) WN = N .s = 0 ( s is perpendicular to N (iv) Wg = m g.s = −mgs sin 150 = −2030 J .

The net work: W NET = WT + W fr + W N + W g = +1.79kJ

V.2.2. ENERGY

In a restricted to mechanics sense, energy can also be defined is a measure of capacity of a system for doing work. Energy has several forms
and is determined by the amount of work a system can do.

W = −E (V.3)

→ A system which performs work has its stored energy diminished and a system on which work is performed (by external forces)
has its stored energy increased
61
Expression (V.3) is also called:”Work-energy equivalence theorem”

Equation (V.3) implies that work and energy have the SI units: Si unit for energy is Joule

E = 1Joule  1J (V.4)

V.3. POWER

Suppose a petrol engine that is used in a crane to lift a load of bricks weighing 9000N from the ground to the top of a building 20m high.
How much work is done?

A = FXd = 9000 NX 20m = 180000 J = 1.8 X 10 5 J

We don’t mention the time needed to perform the work whether the crane lifts the bricks through 20m in 1h or in 10h it still performs
5
the same work: 1.8 X 10 J
But we can discuss the rate at which the work is performed: how fast? That rate defines the power

Definition: “Power is the rate at which the work is performed which energy is transformed from one form to another”

dW
P= (V.5)
dt

P = W  = 1Joule = 1Watt = 1W


t  1sec ond
The SI unit of power (V.6)

The unit of power was named in the honour of James Watt*

An out-system unit of energy and work is the kilowatt. hour: the work done by a force having a power of 1kW during 1hour :

1kW .h = 1000WX 3600s = 3.6 X 10 6 J (V.7)

*James Watt, FRS, FRSE (19 January 1736 – 25 August 1819) was a Scottish inventor and mechanical engineer whose improvements to
the Newcomen steam engine were fundamental to the changes brought by the Industrial Revolution in both his native Great Britain and the
rest of the world

V.4. KINETIC AND POTENTIAL ENERGY

In mechanics, one can distinguish two types of energy: kinetic and potential energy

V.4.1. KINETIC ENERGY

Kinetic energy is energy possessed by a body by virtue of its motion. It is equal to the work that will be required to bring the body to rest

Consider a body B with velocity v and mass m . We suppose that the body is brought to rest by collisions with a stationary body C which
will be deformed. During the process, B acts on C with a certain force F and at a small portion of distance d S , an elementary work

W is accomplished: W = F  d S
According to the Newton’s third law, on body B will act a force −F which causes the change in the velocity of the body:

dv dv dS
−F =m → W = −m dS = −m dv = −mvdv
dt dt dt

62
0
v2
The work accomplished by B before it stops completely is: W = − mvdv = m or
v
2

1 2
K= mv (V.8)
2
→ Kinetic energy of a body in translational motion equals the half of the product of its mass and the square of its velocity

→ Kinetic energy cannot be a negative quantity

→ A moving body trades its kinetic energy into work it performs when it comes to rest

Kinetic energy obeys the law of superposition: kinetic energy of a system of n material particles equals the sum of kinetic energies of all
particles:

N
1
K =  mi v i
2
(V.9)
i =1 2

V.4.2. POTENTIAL ENERGY AND LAW OF CONSERVATION OF TOTAL MECHANICAL ENERGY

Suppose a stone thrown vertically upwards with initial velocity v 0 . Experiment shows that its velocity decreases as the height increases. At a
maximum height, its velocity and kinetic energy are zero and it begins falling down with acceleration

It is easy to prove that the velocity with which it reaches the ground equals the initial velocity v 0 but opposite in direction

1 1
= =
2 2
When it was thrown up it had kinetic energy: K mv0 when it reaches the ground it has kinetic energy: K mv0
2 2
However when it was at maximum height its kinetic energy was zero

→ We have to assume that at maximum height, the body has another kind of energy which depends only on the height (nothing can be
created from nothing!!!).

The proof is that the stone is able to fall under action of force of gravity and perform work. This energy depends only on the configuration of
the system body - Earth and is called “gravitational potential energy”

When the stone falls, its velocity and kinetic energy increases but its height decreases:

→ When the stone goes up, its kinetic energy transforms into potential energy. When it goes down its potential energy transforms
back into kinetic

The sum of kinetic and potential energy is called ”total mechanical energy” and if we neglect friction it is a constant

E = K + U = const (V.10)

Expression (V.10) is called “law of conservation of total mechanical energy” and is one of main laws of nature

The expression for potential energy is:

x
U = −W = −  F ( x )dx (V.11)
x0

63
dU (x )
F (x ) = − (V.12)
dx
x
U ( x ) = −  F (x ) + U (x0 ) (V.13),
x0

where x 0 is a certain reference position of the body

The potential energy when the body is at reference position U (x0 ) is usually taken arbitrary equal to zero

For the force of gravity, if we choose the direction of coordinate upwards:

h h
F (h ) = − m g = const → U (h ) = − F (h )d h + U (0) = − m gd h + U (0) = mgh + U 0
0 0

If we take U = 0 for h = 0

U (h) = mgh (V.14)

Expression (V.14) gives the potential energy of a body located at height h from the ground (Earth’s surface)
V.5. CONSERVATIVE AND DISSIPATIVE SYSTEMS

Systems in which the principle of conservation of total mechanical energy applies are called “conservative”

Forces that act on these systems are called “conservative” forces

→ Force of gravity is a conservative force

A second example of conservative force is that exerted by an elastic spring on a body of mass m attached to it and acting on a horizontal
frictionless surface:

If x = 0 is the position of the end of the spring when it is not deformed (at equilibrium):
x x
The elastic force is: F = −k x → U (x ) = −  F (x )dx + U (0) = − (− kx)dx + U 0
0 0

x
U (0) = 0 , U (x ) = − (− kx)dx = kx2
1
If we choose
0
2

U (x ) =
1 2
kx (V.15)
2
Equation (V.15) is the expression for elastic potential energy and it is equal to the work done by the deformed spring to recover its
undeformed or its equilibrium state

1 2 1 2 1
E= mv + kx = mv0 ,
2
The total energy, sum of kinetic and potential energy, will be:
2 2 2
64
where v0 is the speed of the spring when x = 0

Fig.V.2. Kinetic and elastic potential energy of a spring

1 2
At the point x m when the speed of the body is zero and all energy is potential: kxm
2
According to the principle of conservation of total mechanical energy: at any time and any position:

1 2 1 2 1 1
kx + mv = kxm = mv0
2 2
(V.16)
2 2 2 2

We can calculate the maximum extension of the spring x m if we know the initial velocity v0 :

m
xm = v0 (V.17)
k
Practical Activity V.2

Example V.2

N
A block of mass 100 g is attached to the end of a spring whose spring constant is k = 40 . The block slides on a horizontal
m
surface for which k fr = 0.1 . The spring is extended by 5cm and then released. (i) Find the work done by the spring up to the
point at which it is compressed by 3cm (ii) Find the net work done on the block up to this point
Solution

(i) From equation (V.15)


2
2
( 2

m
)
WSP = − k x f − xi = −20 (9 X 10 − 4 − 25 X 10 − 4 ) = −0.032 J
1 N

65
(ii) The work done by force of friction, F fr = − k fr mg → W fr = − F fr s = (0.098 N )(
. 0.08m ) = −0.0078 J

The net work done on the block is WNET = WSP + W fr = +0.032 − 0.0078 = 0.0242 J

V.5.1. NON-CONSERVATIVE OR DISSIPATIVE FORCES

An example of a non-conservative force is friction force: Friction force is always directed opposite the velocity of the body → The work
of friction force is always negative: it is a dissipative force that decreases the total mechanical energy budget in a system.

Summary

Energy is th universal measure of motion of matter. It qualitatively and quantitatively accounts for transformations of one type of motion to
another.

Work is one of the forms of energy.

The work done by a constant force F = Fx i + Fy j + Fz k when its point of application undergoes a displacement

s = xi + y j + z k is:

W = F.s = F.s cos W = Fx x + Fy yFz z

In one dimension the work done by a variable force is

B
W A→ B =  Fx dx
A

The work-energy theorem relates the work done by the net force on a particle to the resulting change in its kinetic energy:

W = −K ,

1 2
where K= mv is the kinetic energy of the particle
2
(1)
= WA→B
( 2)
The work done by a conservative force is independent of the path taken between the points A and B: WA→B or equivalently
the work done by a conservative force in any close loop is zero. This is called the principal of path independence

Potential energy is energy associated with the mutual position of interacting particles in a system. It belongs to the system. Potential energy
can be defined only for a conservative force. The change in potential energy between two points is related to the work done by the associated
conservative force

U = −W
The potential energy is defined relative to a reference configuration of particles in the system for which the potential energy is taken as equal
to zero

Gravitational potential energy: U G = mgh for g = const

1 2
Elastic potential energy: U EL = kx
2

Total mechanical energy: E = K +U


66
Principle of conservation of total mechanical energy: E = K + U = const
Self-test

V.1. True or false: If the kinetic energy of a body is zero, then the net force on it is zero Explain your response

V.2. Do devices also called “simple machines” such as levers, pulleys and gears, which allow us to use smaller forces also save us
work? Explain

V.3. You lift a box from the ground and place it on a table. Does the amount of work you do on the box depend on how fast you lift it?

V.4. If the net work done on a particle is zero, what can we conclude about the following: (i) its acceleration , (ii) its velocty

V.5. True or false: (i) Conservation of mechanical energy does not apply to nonconservative forces (ii) The net force acting
on a ball being lifted at constant velocity is zero: hence, its energy is conserved

V.6. A body is dropped from rest. Sketch the variation of the kinetic energy and potential energy as functions of (i) height (ii) time

H vmax
V.6. True or false: If a body is dropped from height H , then at : (i) velocity v= (ii) kinetic
4 2
K max K 3
energy K = (iii) =
4 U 4
V.7. When you descend at constant speed in an elevator, what happens to your lost potential energy?

Practical Activity V.3

Exercises V.1

1. The coefficient of kinetic friction between a 20 − kg box and the floor is 0.40 . How much work does a pulling force do on the
box in pulling it 8.0m across the floor at constant speed? The pulling force is directed 37 0 above the horizontal

2. 3T travels uphill at angle of 15 0 to the horizontal. What work the engine performs to move the truck
A small truck of mass
m
50m if the truck moves with acceleration 0.2 2 ? Coefficient of kinetic friction is 0.1
s

m
3. A crate of mass 50kg slides down a 30 0 incline. The crate’s acceleration is 2.0 , and the incline is 10m long.
s2
(a) What is the kinetic energy of the crate as it reaches the bottom of the incline?

(b) How much work is spent in overcoming friction?

4. What is the magnitude of the frictional force that acts on the crate as it slides down the incline?

km
5. In Fig.IV.3 a runaway 5T -truck with failed brakes is moving downgrade at 130 just before the driver steers up the truck up
h
0
a frictionless emergency escape ramp with an inclination of 15 . (a) What minimum length L must have the ramp if the truck is
to stop instantly? Does the minimum length increase, decrease or remain the same if (b) the truck’s mass is decreased and (c) the
speed is decreased

67
Fig.V.3. Tutorial V.4.

6. In Fig. IV.5, a 12 kg -block is released from rest on a 30 0 frictionless incline. Below the block is a spring that can be compressed
2.0cm by a force of 270 N . The block stops momentarily when it compresses the spring by 5.5cm . (a) How far does the block
move down the incline from its rest position to this stopping point? (b) What is the speed of the block just as it touches the spring?

Fig. V.4. Tutorial IV.5

1. A locomotive pulls a train of mass 500T along a horizontal track. The power of the locomotive is 1400HP . Coefficient of
km
kinetic friction is 0.01 . Find the acceleration of the train at the moment when its speed is 36
h

km
2. A truck of mass 3T when traveling along a 3 − km horizontal stretch of road increases its speed from 36 to 72 . The
h
coefficient of kinetic friction is 0.01 . Find the work done and average power of the truck’s engine during the journey.

g
3. A helicopter lifts a 72kg astronaut 15m vertically from the ocean by means of a cable. The acceleration of the astronaut is .
10
How much work is done on the astronaut by (a) the force from the helicopter and (b) the gravitational force on her? What are (c)
the kinetic energy and (d) the speed of the astronaut just before she reaches the helicopter?

4. A steel ball of mass 0.500kg is fastened to a cord that is 70.0cm long and fixed at the far end. The ball is then released when
the cord is horizontal. Fig.V.12. At the bottom of its path the ball strikes a 2.5kg steel block initially at rest on a frictionless
surface. The collision is elastic. Find (a) the speed of the ball and (b) the speed of the block, both just after the collision.

68
Fig.V.5. Problem 4

5. You push a 2.0kg block against a horizontal spring, compressing the spring by 15cm . Then you release the block, and the
N
spring sends it sliding across the tabletop. It stops 75cm from where you released it. The spring constant is 200 . What is the
m
coefficient of kinetic friction between the block and the table?

69

You might also like